Administración     

Olimpiadas de Matemáticas
Página de preparación y problemas

OME Local
OME Nacional
OIM
OME Andalucía
Retos UJA
Selector
La base de datos contiene 1154 problemas y 775 soluciones.

IX Olimpiada Iberoamericana de Matemáticas — 1994

Sesión 1 —  Fortaleza (Brasil), 20 de septiembre de 1994

Problema 507
Se dice que un número natural $n$ es sensato si existe un entero $r$, con $1\lt r \lt n-1$, tal que la representación de $n$ en base $r$ tiene todas sus cifras iguales. Por ejemplo, $62$ y $15$ son sensatos ya que $62 = 222_{(5)}$ y $15 = 33_{(4)}$ . Demostrar que $1993$ no es sensato, pero que $1994$ sí lo es.
pistasolución 1info
Pista. Observa que $1993$ es primo mientras que $1994$ no lo es.
Solución. Que el número $n$ sea sensato equivale a que \[n=a(1+r+r^2+\ldots+r^k)\] para ciertos $a,r\in\mathbb{N}$ tales que $1\lt r\lt n-1$ y $1\leq a\leq r-1$. El caso de $1994=2\cdot 997$ es obvio ya que basta tomar $a=2$, $r=996$ y $k=1$, lo que nos da la representación $1994=22_{(996)}$.

Como $1993$ es primo, no vale el truco anterior y tenemos que necesariamente $a=1$. Por tanto, $1993$ será sensato cuando podamos encontrar $r,k\geq 2$ tales que \[1993=1+r+\ldots+r^k.\] Esta ecuación nos dice además que $r$ debe ser un divisor de $1992=2^3\cdot 3\cdot 81$. No obstante, $r$ no puede ser múltiplo de $81$ ya que $81^2\gt 1993$, lo que nos deja sólo las posibilidades $r\in\{2,3,4,6,8,12,24\}$. En realidad, se pueden probar una a una sin perder demasiado tiempo, pero una opción sin tanto cálculo es observar que $$1993=1+r+\ldots+r^k=\frac{r^{k+1}-1}{r-1}\ \Leftrightarrow\ 1+1993(r-1)=r^{k+1},$$ luego sólo tenemos que calcular $1+1993(r-1)$ y ver si es potencia de $r$ o no:

  • Para $r=2$, tenemos que $1+1993(r-1)=1994=2\cdot 997$ no es potencia de $2$.
  • Para $r=3$, tenemos que $1+1993(r-1)=3987=9\cdot 443$ no es potencia de $3$.
  • Para $r=4$, tenemos que $1+1993(r-1)=5980$ es múltiplo de $5$ y no es potencia de $4$.
  • Para $r=6$, tenemos que $1+1993(r-1)=9966$ es múltiplo de $11$ y no es potencia de $6$.
  • Para $r=8$, tenemos que $1+1993(r-1)=13952=2^7\cdot 109$ no es potencia de $8$
  • Para $r=12$, tenemos que $1+1993(r-1)=21924$ es múltiplo de $7$ y no es potencia de $12$.
  • Para $r=24$, tenemos que $1+1993(r-1)=45840$ es múltiplo de $5$ y no es potencia de $24$.
Concluimos que $1993$ no es sensato.

Si crees que el enunciado contiene un error o imprecisión o bien crees que la información sobre la procedencia del problema es incorrecta, puedes notificarlo usando los siguientes botones:
Informar de error en enunciado Informar de procedencia del problema
Problema 508
Sea $ABCD$ un cuadrilátero inscrito en una circunferencia tal que existe una semicircunferencia con centro en $AB$ y tangente a los otros tres lados del cuadrilátero.
  1. Demostrar que $AB=AD+BC$.
  2. Calcular, en función de $x = AB$ e $y = CD$, el área máxima que puede alcanzar un cuadrilátero en estas condiciones.
Sin pistas
Sin soluciones
info
Si crees que el enunciado contiene un error o imprecisión o bien crees que la información sobre la procedencia del problema es incorrecta, puedes notificarlo usando los siguientes botones:
Informar de error en enunciado Informar de procedencia del problema
Problema 509
En cada casilla de un tablero $n\times n$ hay una lámpara. Al ser tocada una lámpara cambian de estado ella misma y todas las que están situadas en la misma fila o columna (las que están encendidas se apagan y las apagadas se encienden). Inicialmente todas están apagadas. Demostrar que siempre es posible, con una sucesión adecuada de toques, que todo el tablero quede encendido, y encontrar, en función de $n$, el mínimo número de toques para que se enciendan todas las lámparas.
Sin pistas
solución 1info
Solución. Si $n$ es impar, podemos tocar todas las lámparas de la primera fila (o de culquier fila o columna) y todas quedan encendidas. En este caso, $n$ es realmente el mínimo número de toques ya que, si sólo tocáramos $n-1$ lámparas o menos, entonces habría una fila y una columna en la que no tocaríamos ninguna lámpara. La casilla intersección de esta fila y columna quedaría apagada.

Si $n$ es par, entonces nuestro objetivo se cumplirá si tocamos las $n^2$ lámparas ya que a cada lámpara le afectan $2n-1$ cambios de estado (en realidad, esto también funciona para $n$ impar pero no es óptimo, como acabamos de ver). Veamos que, si no tocamos todas las lámparas, entonces habrá alguna que quede apagada, lo que probará que $n^2$ es óptimo en este caso (observemos que no tiene sentido tocar una misma lámpara dos veces).

Si crees que el enunciado contiene un error o imprecisión o bien crees que la información sobre la procedencia del problema es incorrecta, puedes notificarlo usando los siguientes botones:
Informar de error en enunciado Informar de procedencia del problema

Sesión 2 —  Fortaleza (Brasil), 21 de septiembre de 1994

Problema 510
Sea $ABC$ un triángulo acutángulo con circunferencia circunscrita $K$. Sea $P$ un punto interior a $K$. Se trazan las rectas $AP$, $BP$ y $CP$ que cortan de nuevo a $K$ en $X$, $Y$ y $Z$, respectivamente. Determinar el punto $P$ para que el triángulo $XYZ$ sea equilátero.
Sin pistas
Sin soluciones
info
Si crees que el enunciado contiene un error o imprecisión o bien crees que la información sobre la procedencia del problema es incorrecta, puedes notificarlo usando los siguientes botones:
Informar de error en enunciado Informar de procedencia del problema
Problema 511
Sean $n$ y $r$ dos enteros positivos. Se desea construir $r$ subconjuntos $A_1, A_2,\ldots,A_r$ de $\{0,1,\ldots,n-1\}$ de exactamente $k$ elementos cada uno y de forma que todo entero $x$ tal que $0\leq x\leq n-1$ se puede escribir como $x=x_1+x_2+···+x_r$ con $x_1\in A_1$, $x_2\in A_2$, ..., $x_r\in A_r$. Hallar el menor valor posible de $k$ en función de $n$ y $r$.
Sin pistas
Sin soluciones
info
Si crees que el enunciado contiene un error o imprecisión o bien crees que la información sobre la procedencia del problema es incorrecta, puedes notificarlo usando los siguientes botones:
Informar de error en enunciado Informar de procedencia del problema
Problema 512
Demostrar que todo número natural $n\leq 2^{1000000}$ puede ser obtenido a partir del $1$ haciendo menos de $1100000$ sumas. Más precisamente, hay una sucesión finita de números naturales $x_0,x_1,\ldots, x_k$ con $k\lt 1100000$, $x_0 = 1$ y $x_k = n$, tal que para cada $i=1,2,\ldots,k$ existen $r, s$ con $0\leq r\lt i$, $0\leq s\lt i$ y $x_i =x_r +x_s$.
pistasolución 1info
Pista. Se puede duplicar un número sumándolo consigo mismo. Trabaja el problema en base $2$.
Solución. Las primeras $2^{12}-2=4094$ sumas las emplearemos en obtener los números desde el $2$ hasta el $2^{12}-1$ sin más que sumar $1$ consigo mismo repetidamente. En base $2$ esto nos da todos los números de $11$ cifras y también el $2^{12}$, que es un uno seguido de once ceros. Veamos cómo usar esto para hallar cualquier número $n$ de $999999$ cifras en base $2$, cifras que se pueden agrupar en $90909$ grupos de once dígitos consecutivos formando números de once cifras $S_1,S_2,\ldots,S_{90909}$, siendo $S_1$ las once cifras más significativas

Como $S_1\leq 4095$, este número se encuentra entre los calculados. Ahora lo sumamos consigo mismo para obtener $2S_1$ y este resultado lo sumamos consigo mismo para obtener $4S_1$ y así sucesivamente hasta obtener $2^{11}S_1$, que es el número $S_1$ seguido de $11$ ceros. Ahora sumamos al resultado $S_2$ y ya tenemos el número $2^{11}S_1+S_2$, que contiene las veintidós cifras más significativas de $n$. Lo sumamos consigo mismo de forma que en otros once pasos obtenemos $2^{22}S_1+2^{11}S_2$ y le sumamos $S_3$. Podemos repetir este proceso para conseguir todas las cifras de $n$. Por cada grupo de once cifras que añadimos, hemos necesitado $12$ sumas (once para multiplicar por $2^{11}$ y una para añadir once nuevos dígitos), lo que hace un total de $4095+12\cdot 90908=1094991\lt 1100000$.

Queda el caso de que $n=2^{1000000}$, pero este número se obtiene con solo $1000000$ sumas sin más que sumar $1$ consigo mismo y luego $2$ consigo mismo, luego $4$ consigo mismo, y así sucesivamente.

Nota. Esta solución funciona haciendo grupos de $11$ (como hemos hecho), de $12$ (se obtiene un máximo de $1091520$ sumas) o $13$ dígitos ($1093291$ sumas). Con grupos de más de $13$ dígitos o de menos de $11$, necesitaríamos en general más de $1100000$ sumas mediante este método.

Si crees que el enunciado contiene un error o imprecisión o bien crees que la información sobre la procedencia del problema es incorrecta, puedes notificarlo usando los siguientes botones:
Informar de error en enunciado Informar de procedencia del problema
José Miguel Manzano © 2010-2024. Esta página ha sido creada mediante software libre